In a class of 110 students there are as many females 18 years to age is 65. further the number of are over le years in 180. A student picked at random from the class. Klhat, is the probability is male or over 18 18 yrs 2. If a fair Con is tassed twice determine the probability exeat exac e exactly () C Tail (1) 2 hoads 3. (f new the probability of having a home family, is planning to howe 6 childien determine ® 3 girls and 13 boys gizli and 3 boys (10) More than 4 boys Ciu) All girls.

Answers

Answer 1

The probability of getting more than 4 boys is 7/64 iii) Getting all girls: There is only one way of having all 6 girls. The probability of getting all girls is 1/64

The given statements can be summarized as: In a class of 110 students, the number of females is equal to the number of males in the age range of 18 years to 65 years.

The number of students over 65 years is 180.

The probability that a student picked at random is male or over 18 years old

The required probability is given by P(Male or Over 18) = P(Male) + P(Over 18) - P(Male and Over 18)

The probability of being male = number of males / total students = (110 - number of females) / 110

The probability of being over 18 = number of students over 18 / total students = (110 - number of students under 18) / 110

The probability of being male and over 18 = number of males over 18 / total students = (110 - number of females) - number of students under 18 / 110

Substituting the given values, we get: P(Male or Over 18) = [(110 - number of females) / 110] + [(110 - number of students under 18) / 110] - [((110 - number of females) - number of students under 18) / 110] = (110 + number of students over 18 - number of females) / 1102.

Probability of tossing a fair coin twice and getting exactly 1 tail and 1 headIf a fair coin is tossed twice, then the possible outcomes are: (H, H), (H, T), (T, H), and (T, T)

There are four possible outcomes and two of them have exactly one head and one tail. Therefore, the required probability is 2/4 = 1/23.

Probability of having a family of 6 children and getting either 3 girls and 3 boys, or more than 4 boys, or all girlsThe total number of ways of having a family of 6 children is 2^6 = 64.

There are three cases as follows:i) Getting 3 girls and 3 boys: The number of ways of choosing 3 girls out of 6 is (6C3) = 20.

The number of ways of choosing 3 boys out of 6 is (6C3) = 20. Therefore, the total number of ways of having 3 girls and 3 boys is (20 × 20) = 400.

The probability of getting 3 girls and 3 boys is 400/64 = 25/4ii) Getting more than 4 boys: There is only one way of having all 6 boys.

The number of ways of having 5 boys is 6C5 = 6.

The total number of ways of having more than 4 boys is (1 + 6) = 7.

The probability of getting more than 4 boys is 7/64 iii)

Getting all girls: There is only one way of having all 6 girls. The probability of getting all girls is 1/64

Know more about probability here:

https://brainly.com/question/25839839

#SPJ11


Related Questions

which of the following is the average rate of change over the interval [−3, 0] for the function g(x) = log2(x 4) − 5?

A. 3/7
B. 7/3
C. 2/3
D. 3/2

Answers

-4 of the following is the average rate of change over the interval [−3, 0] for the function g(x) = log2(x 4) − 5.

To find the average rate of change over the interval [-3,0] for the function

g(x) = log2(x4) - 5, we can use the formula as shown below;

Average Rate of Change = (g(b) - g(a)) / (b - a)

Where 'b' and 'a' represent the endpoints of the interval [-3,0].

We can therefore plug in these values into the formula as shown below;

Average Rate of Change = [g(0) - g(-3)] / (0 - (-3))

We can then calculate g(0) and g(-3) as follows;

g(0) = log2(04) - 5 = -5g(-3) = log2((-3)4) - 5 = 7

Therefore, the average rate of change over the interval [-3,0] is;

Average Rate of Change = (g(0) - g(-3)) / (0 - (-3)) = (-5 - 7) / (0 + 3) = -12 / 3 = -4

So, the answer is not given in the options provided. Therefore, the correct answer is none of the options.

To know more about  function visit:

https://brainly.com/question/30721594

#SPJ1

Answer:

2/3

Step-by-step explanation:

trust and believe

In general, what is the logic of the statistical test of hypothesis? Collect data, and find the value of the test statistic. If the probability of the null hypothesis given the value of the test stati

Answers

The logic of the statistical test of hypothesis is to collect data and then find the value of the test statistic. If the probability of the null hypothesis given the value of the test statistic is very low, then we can reject the null hypothesis and accept the alternative hypothesis.

The process of testing a hypothesis involves the following steps:Step 1: Collect data related to the problem. This data could be collected through various means like surveys, experiments, or observational studies.Step 2: Define the null and alternative hypotheses. Step 4: Find the value of the test statistic using the collected data. The test statistic is calculated based on the sample data collected and reflects the difference between the sample means, proportions or variances.Step 5: Calculate the p-value. The p-value is the probability of observing a test statistic as extreme or more extreme than the one calculated, assuming the null hypothesis is true.Step 6: Compare the p-value with the significance level (alpha). If the p-value is less than alpha, then we reject the null hypothesis. If the p-value is greater than alpha, then we fail to reject the null hypothesis.

The logic of the statistical test of hypothesis is based on the concept of probability. Probability is a measure of the likelihood of an event occurring. In the context of statistical hypothesis testing, we use probability to determine the likelihood of obtaining a particular test statistic if the null hypothesis is true.Statistical hypothesis testing involves making a decision based on the probability of obtaining a particular test statistic. The null hypothesis is a statement of no difference between two groups or variables.  If the p-value is not very low, then there is not enough evidence to reject the null hypothesis.Finally, we compare the p-value with the significance level (alpha). The significance level is the maximum probability of rejecting the null hypothesis when it is actually true. If the p-value is less than alpha, then we reject the null hypothesis. If the p-value is greater than alpha, then we fail to reject the null hypothesis.In conclusion, the logic of the statistical test of hypothesis involves collecting data, defining the null and alternative hypotheses, choosing an appropriate test, calculating the test statistic, calculating the p-value, and comparing the p-value with the significance level. If the p-value is less than the significance level, then we reject the null hypothesis and accept the alternative hypothesis.

To know more about statistical test  visit :-

https://brainly.com/question/31746962

#SPJ11








(5 points) Evaluate the double integral -1.3 (3x - y) dA, where R is the region in the first quadrant enclosed by the circle x² + y² = 4 and the lines x = 0 and y = x, by changing to polar coordinat

Answers

The solution of the given double integral is(3/4) - (1/4)π/4 + (3/5)π/4³.

Given,The double integral -1.3 (3x - y) dA,

where R is the region in the first quadrant enclosed by the circle x² + y² = 4 and the lines x = 0 and y = x, by changing to polar coordinate.

We know that the polar coordinate is defined by the radius r and the angle θ.

We also know that the radius is given by:r² = x² + y²We can convert the double integral to a polar coordinate as follows:

First, we need to find the limits of integration in polar coordinates. Since R is in the first quadrant, both the radius and the angle are positive. Therefore, we have:

r: 0 to 2θ: 0 to π/4

The limits of integration in the x-y plane are given by the equation of the circle x² + y² = 4 and the lines x = 0 and y = x.

In polar coordinates, these equations are:r² = 4 (equation of circle)r sin θ = r cos θ (equation of line y = x)

Simplifying the second equation:

r = tan θThe region R is enclosed by these curves, so the limits of integration for r are:

r = 0 to tan θ

Now, we can change the double integral to polar coordinates as follows:

dA = r dr dθ

The function 3x - y is converted to polar coordinates as follows:

x = r cos θy = r sin θ

Therefore, the double integral becomes:

I = ∫∫R (3x - y) dA= ∫θ=0^(π/4) ∫r

=0^(tanθ) [(3r cos θ) - (r sin θ)] r dr dθ

= ∫θ=0^(π/4) ∫r

=0^(tanθ) (3r² cos θ - r³ sin θ) dr dθ

Now, we can integrate the inner integral with respect to r and the outer integral with respect to θ.I = ∫θ=0^(π/4) [(3/3) tan³θ cos θ - (1/4) tan⁴θ sin θ] dθ= (3/4) - (1/4)π/4 + (3/5)π/4³

The solution of the given double integral is(3/4) - (1/4)π/4 + (3/5)π/4³.

To know more about Quadrant visit:

https://brainly.com/question/29298581

#SPJ11


differential equations
(d) If the Wronskian of f, g is W(f.g) = 7, then W(4f+g,f+2g)= 49

Answers

The given problem involves the Wronskian, which is a determinant used in the study of differential equations. In this case, we are given the Wronskian of two functions, f and g, as 7. The problem asks us to determine the Wronskian of two new functions, 4f+g and f+2g, and we are given that this value is equal to 49.

To understand the solution, let's start with the definition of the Wronskian. The Wronskian of two functions, say f and g, denoted as W(f,g), is given by the determinant of the matrix formed by the derivatives of these functions. In this case, we are not given the explicit forms of f and g, but we know that W(f,g) is equal to 7.

Now, to find the Wronskian of 4f+g and f+2g, denoted as W(4f+g,f+2g), we can use some properties of determinants. One property states that if we multiply a row (or column) of a matrix by a constant, the determinant of the resulting matrix is equal to the constant multiplied by the determinant of the original matrix. Applying this property, we can rewrite the Wronskian as W(4f+g,f+2g) = (4*1+1*2)W(f,g) = 9W(f,g).

Since we know that W(f,g) = 7, we can substitute this value into the expression to find W(4f+g,f+2g) = 9W(f,g) = 9*7 = 63. Therefore, the Wronskian of 4f+g and f+2g is 63, not 49 as initially stated in the problem.

In summary, the given problem involved finding the Wronskian of two functions based on a given Wronskian value. However, the solution revealed that there was an error in the problem statement, as the correct Wronskian of 4f+g and f+2g is 63, not 49. The explanation involved using the properties of determinants to manipulate the expression and arrive at the final result.

Learn more about determinant here : brainly.com/question/29898039

#SPJ11

Use the following information for problems 8 and 9. Suppose that variables X and Y are both continuous random variables. The mean of X is a, and the standard deviation of X is b. The mean of Y is c, and the standard deviation of Y is d. Find the mean of X+Y. O (a + c)/2 O a + c O a-c O a.c

Answers

If given the continuous random variables, X and Y, the mean of X + Y would be B. a + c

How to obtain the mean of two variables

To obtain the mean of two variables, we have to take the sum of their means. This is slightly different from simplet numbers where we add all the numbers and divide by the totality of them all.

For random variables as indicated in the question above, given mean of X as a and the mean of Y as c, the mean of X + Y can be obtained by summing the two means. So, option B is correct.

Learn more about continuous random variables here:

https://brainly.com/question/27761372

#SPJ1

El sonar de un barco de salvamento localiza los restos de un naufragio en un ángulo de depresión de 30°. Un buzo es bajado 40 metros hasta el fondo del mar. ¿Cuánto necesita avanzar el buzo por el fondo para encontrar los restos del naufragio?

Answers

The diver has to travel approximately 69.28 meters to reach the wreckage of the ship.

The problem involves finding the horizontal distance that a diver has to cover to reach the wreckage of a ship after a rescue boat detects the signal at an angle of depression of 30°. The diver descends 40 meters to the seafloor.

The concept of trigonometry is useful in solving the problem. Here are the steps to solve the problem:

Step 1: Draw a diagram that represents the problem.

Step 2: Let the horizontal distance that the diver has to travel be "d".

Step 3: Let the angle of depression be "θ". From the diagram, we can see that tan θ = d / 40m.

Step 4: Substitute the value of θ and solve for "d".tan 30° = d / 40m1 / √3 = d / 40m√3d = 40m√3d ≈ 69.28 meters

To learn more about : meters

https://brainly.com/question/30153116

#SPJ8

Find a matrix K such that AKB = C given that A = [ 1 4], B = [5 0 0], C = [130 60 -60]
[-2 3] [0 4 -4] [70 12 -12]
[ 1 -2] [-50 -12 12]
K =

Answers

The matrix K that satisfies AKB = C is K = [2 3; -1 2; -3 4].

To find the matrix K, we need to solve the equation AKB = C. Since A has dimensions 2x2, B has dimensions 2x3, and C has dimensions 3x3, the resulting matrix after multiplying AKB must also have dimensions 2x3.

Let K be a matrix with dimensions 2x3, where each entry is represented as K = [k1 k2 k3; k4 k5 k6].

Multiplying AKB, we get:

AKB = [1 4][k1 k2 k3; k4 k5 k6][5 0 0; 0 4 -4]

   = [1 4][5k1 4k2 -4k3; 5k4 4k5 -4k6]

   = [5k1 + 20k4 4k1 + 16k5 - 16k6; 5k4 + 20k2 4k4 + 16k5 - 16k6].

Comparing the resulting matrix with C, we can set up the following equations:

5k1 + 20k4 = 130

4k1 + 16k5 - 16k6 = 60

5k4 + 20k2 = -2

4k4 + 16k5 - 16k6 = 3

4k5 - 4k6 = -60

Solving these equations, we find k1 = 2, k2 = 3, k4 = -1, k5 = 2, and k6 = 4. Therefore, K = [2 3; -1 2; -3 4].

Learn more about dimensions here: brainly.com/question/31156956

#SPJ11

Janet earned $78,000 last year. Tax rate earned on the first $20,000 is 15%; 25% on the next $25,000 and 30% for the remainder of income. What was the amount of tax paid?

Answers

Janet paid a total of $19,150 in taxes. She owed $3,000 on the first $20,000 of income at a 15% tax rate, $6,250 on the next $25,000 at a 25% tax rate, and $9,900 on the remaining $33,000 of income at a 30% tax rate.

The amount of tax paid by Janet, we need to determine the tax owed on each portion of her income and then sum them up.

Step 1: Calculate the tax owed on the first $20,000, taxed at a rate of 15%: $20,000 * 0.15 = $3,000.

Step 2: Calculate the tax owed on the next $25,000, taxed at a rate of 25%: $25,000 * 0.25 = $6,250.

Step 3: Calculate the remaining income after considering the first $45,000: $78,000 - $45,000 = $33,000.

Step 4: Calculate the tax owed on the remaining income, taxed at a rate of 30%: $33,000 * 0.30 = $9,900.

Step 5: Sum up the tax owed on each portion of income: $3,000 + $6,250 + $9,900 = $19,150.

Therefore, the amount of tax paid by Janet is $19,150.

Learn more about tax rate : brainly.com/question/30629449

#SPJ11

A microchip manufacturer controls the quality of its products by inspecting a batch of 100 microchips, taking a sample of 20, if at least 3 of these are defective, the entire batch will be rejected. The lot contains 12 defective microchips.
a) What is the probability of rejecting the lot?
b) What is the probability of not finding defective microchips?
c) Find the expected value of the probability distribution E(x) and the standard deviation.

Answers

a) Probability of rejecting the lot is 0.936.

b) Probability of not finding defective microchips is 0.318.

c) Expected value of the probability distribution E(x) is 2.4 and the standard deviation is 0.49.

a) Probability of rejecting the lot:P(at least 3 of the 20 are defective) = P(3 defectives) + P(4 defectives) + P(5 defectives) + P(6 defectives) + P(7 defectives) + P(8 defectives) + P(9 defectives) + P(10 defectives) + P(11 defectives) + P(12 defectives)= (12C3*88C17 + 12C4*88C16 + 12C5*88C15 + 12C6*88C14 + 12C7*88C13 + 12C8*88C12 + 12C9*88C11 + 12C10*88C10 + 12C11*88C9 + 12C12*88C8)/100C20= 0.936b)

Probability of not finding defective microchips:P(0 defective) + P(1 defective) + P(2 defective) = (12C0*88C20 + 12C1*88C19 + 12C2*88C18)/100C20= 0.318c)

Expected value of the probability distribution E(x):mean = E(x) = n * p = 20 * 0.12 = 2.4

Standard deviation: SD = sqrt(np(1-p))= sqrt(20 * 0.12 * (1-0.12)) = 0.49

Know more about Probability here:

https://brainly.com/question/25839839

#SPJ11

Consider a four-step serial process with processing times given in the list below. There is one machine at each step of the process and this is a machine-paced process.

Step 1: 26 minutes per unit

Step 2: 16 minutes per unit

Step 3: 23 minutes per unit

Step 4: 26 minutes per unit

Assuming that the process starts out empty, how long will it take (in hours) to complete a batch of 91 units? (Do not round intermediate calculations. Round your answer to the nearest hour)

Answers

The time required to complete a batch of 91 units is approximately 138 hours.

To calculate the total time required to complete a batch of 91 units in a four-step serial process, we need to consider the processing times for each step and add them up.

Step 1 takes 26 minutes per unit, so for 91 units, it would take 26 * 91 = 2366 minutes.

Step 2 takes 16 minutes per unit, so for 91 units, it would take 16 * 91 = 1456 minutes.

Step 3 takes 23 minutes per unit, so for 91 units, it would take 23 * 91 = 2093 minutes.

Step 4 takes 26 minutes per unit, so for 91 units, it would take 26 * 91 = 2366 minutes.

To find the total time, we add up the individual step times: 2366 + 1456 + 2093 + 2366 = 8281 minutes.

To convert minutes to hours, we divide the total time by 60: 8281 / 60 = 138.0167 hours. Rounding to the nearest hour, the time required to complete a batch of 91 units is approximately 138 hours.

LEARN MORE ABOUT time here: brainly.com/question/15356513

#SPJ11

A1. Consider the initial value problem comprising the ODE
dy/dx= 1 / y²-1
subject to the initial condition.
y(X) = Y,
where X and Y are known constants.
(i) Without solving the problem, decide if (and under what conditions) this initial value problem is guaranteed to have a unique solution. If it does, is the existence of that solution necessarily guaranteed for all values of x?
(ii) Determine the ODE's isoclines, sketch its direction field in the range x € [-3,3] and y € [-3,3]. then sketch a few representative integral curves. [Hint: You should not have to draw the direction field along more than five equally-spaced isoclines, say.] Discuss briefly how the plot of the solution curves relates to the existence and uniqueness results from part (i).
(iii) Find the general solution of the ODE, then apply the initial condition y(0) = 0. You may leave the solution in implicit form.

Answers

C = ±1 and the general solution becomes: y = ±sqrt((dy/dx)⁻¹ + 1) = ±sqrt(x² + 1) The above solution can be obtained in implicit form.

Given differential equation is dy/dx = 1/(y² - 1)

Given initial condition is y(x) = y, where x and y are known constants.

(i) To check whether the given initial value problem has a unique solution or not, we need to check the existence and uniqueness theorem which states that:

If f(x,y) and ∂f/∂y are continuous in a rectangle a < x < b and c < y < d containing the point (x₀,y₀), then there exists a unique solution y(x) of the initial value problem dy/dx = f(x,y), y(x₀) = y₀, that exists on the interval [α,β] with α < x₀ < β such that (x,y) ∈ R and y ∈ [c,d].

Here, f(x,y) = 1/(y² - 1) and ∂f/∂y = -2y/(y² - 1)² are continuous functions.

Therefore, the given initial value problem has a unique solution under the condition |y| > 1 or |y| < 1. This solution is guaranteed only on an interval that contains x₀.

That means, we can't extend the solution to the entire domain.

(ii) Isoclines:Let k be a constant, then the isocline can be defined as:dy/dx = k, which represents the set of points (x,y) such that dy/dx = k. Hence, we can obtain the isocline for the given differential equation as follows:1/(y² - 1) = k⇒ y² - 1 = 1/k⇒ y² = 1 + 1/kThe above equation represents the isocline. We can draw this curve by selecting different values of k.

The direction field in the range x ∈ [-3,3] and y ∈ [-3,3] can be obtained by drawing the tangent to the isocline curve at each point.

A few representative integral curves are drawn as follows:

From the above plot, we can observe that the solution curves don't exist for all values of x. It means the solution exists only on an interval that contains the given initial point.

(iii) We can solve the given differential equation as follows:dy/dx = 1/(y² - 1)⇒ y² - 1 = (dy/dx)⁻¹⇒ y² = (dy/dx)⁻¹ + 1⇒ y = ±sqrt((dy/dx)⁻¹ + 1)

The above equation represents the general solution of the given differential equation.

Now, we can apply the initial condition y(0) = 0 to determine the constant.

When x = 0, y = 0. Therefore, C = ±1 and the general solution becomes:y = ±sqrt((dy/dx)⁻¹ + 1) = ±sqrt(x² + 1)The above solution can be obtained in implicit form.

Know more about implicit form here:

https://brainly.com/question/30487261

#SPJ11

4. A particle moves along the x-axis in such a way that its position at time t for t≥ 0 is given by s(t): 1/1 t³ - 3t² +8t. = 3 A. Find the position of the particle at time t = 3. (1 point) B. Show that at time t = 0, the particle is moving to the right. (2 points) C. Find all values of t for which the particle is moving to the left. (2 points) D. What is the total distance the particle travels from t = 0 to t = 4? (4 points)

Answers

The total distance traveled by the particle is 48.

The given function is s(t): (1/1)t³ - 3t² +8t The position of the particle at time t = 3 is given as follows.

Substitute the value of t = 3 in the given function. s(3) = (1/1)(3)³ - 3(3)² +8(3)s(3) = 27 - 27 + 24s(3) = 24 The position of the particle at time t = 3 is 24. Therefore, option A is correct. The velocity of the particle can be found as follows. The derivative of the function s(t) gives the velocity of the particle. s(t) = (1/1)t³ - 3t² +8ts'(t) = d/dt(s(t))s'(t) = d/dt((1/1)t³) - d/dt(3t²) + d/dt(8t)s'(t) = 3t² - 6t + 8

At time t = 0,s'(0) = 3(0)² - 6(0) + 8s'(0) = 8If s'(0) > 0, then the particle is moving to the right.    At time t = 0, the velocity of the particle is s'(0) = 8, which is greater than 0.

Therefore, the particle is moving to the right at t = 0. A particle moving to the left means its velocity is negative. Therefore, we need to find the values of t where the velocity s'(t) is negative. Therefore, solve the inequality s'(t) < 0 for t.3t² - 6t + 8 < 0t² - 2t + 8/3 < 0Solve the above inequality using the quadratic formula.t = (2 ± sqrt(2² - 4(1)(8/3))) / 2(1)t = (2 ± sqrt(-8/3)) / 2t = 1 ± (2/3)iThe roots are complex and have no real solution.

Therefore, the particle is not moving to the left at any time.  

Total distance traveled by the particle from t = 0 to t = 4 can be found as follows. The displacement of the particle from t = 0 to t = 4 can be found by evaluating s(4) - s(0).s(4) = (1/1)(4)³ - 3(4)² +8(4)s(4) = 64 - 48 + 32s(4) = 48s(0) = (1/1)(0)³ - 3(0)² +8(0)s(0) = 0 - 0 + 0s(0) = 0Displacement = s(4) - s(0)Displacement = 48 - 0Displacement = 48

The displacement of the particle is 48.

To know more about complex visit:

https://brainly.com/question/20566728

#SPJ11

Given that a particle moves along the x-axis in such a way that its position at time t for t≥ 0 is given by s(t): 1/1 t³ - 3t² +8t.

A. The position of the particle at time t = 3, s(t) = (1/1) t³ - 3t² +8t is 24 units.

B. It is showed that at t = 0, the particle is moving to the right.

C. The particle moves to the right for all values of t.

D. The total distance the particle travels from t = 0 to t = 4 is  (8 + 24√3)/3 units.

A. To find the position of the particle at time t = 3, s(t) = (1/1) t³ - 3t² +8t.

∴ s(3) = (1/1) (3)³ - 3(3)² +8(3)

∴ s(3) = 27 - 27 + 24

∴ s(3) = 24 units

B. To show that at time t = 0, the particle is moving to the right.

v(t) = s'(t) = 3t² - 6t + 8

∴ v(0) = 3(0)² - 6(0) + 8 = 8 units per second (to the right)

C. Find all values of t for which the particle is moving to the left.

The velocity of the particle is given by v(t) = s'(t) = 3t² - 6t + 8.

For the particle to move to the left, v(t) must be negative.

3t² - 6t + 8 < 0⇒ t² - 2t + 8/3 < 0

The discriminant of the quadratic t² - 2t + 8/3 is (-2)² - 4(1)(8/3) = -8/3.

Since the discriminant is negative, the inequality t² - 2t + 8/3 < 0 has no real solutions.

Therefore, the particle moves to the right for all values of t.

D. To find the total distance the particle travels from t = 0 to t = 4.

The distance the particle travels from t = 0 to t = 4 is given by

d = ∫₀⁴ |s'(t)| dt= ∫₀⁴ |3t² - 6t + 8| dt.

The velocity 3t² - 6t + 8 changes sign at the roots of the quadratic

3t² - 6t + 8 = 0⇒ t = (6 ± √16)/6= 1 ± 1/√3

On the interval 0 ≤ t ≤ 1 - 1/√3,3t² - 6t + 8 > 0.

On the interval 1 - 1/√3 ≤ t ≤ 1 + 1/√3,3t² - 6t + 8 < 0.

On the interval 1 + 1/√3 ≤ t ≤ 4,3t² - 6t + 8 > 0.

∴ d = ∫₀^(1 - 1/√3) (3t² - 6t + 8) dt - ∫^(1 + 1/√3)_(1 - 1/√3) (3t² - 6t + 8) dt + ∫^(4)_^(1 + 1/√3) (3t² - 6t + 8) dt

= 8/3 - (32/3)/√3 + (104/3)/√3 - (8/3)/√3 + (56/3)

= 8/3 + (72/3)/√3

= (8 + 24√3)/3 units (correct to 2 decimal places).

To know more about the distance, visit:

https://brainly.com/question/30035408

#SPJ11

Assume that x has a normal distribution with the specified mean
and standard deviation. Find the indicated probability. (Round your
answer to four decimal places.)
= 42; = 14
P(50 ≤ x ≤

Answers

We have given mean = μ = 42 Standard deviation = σ = 14Using the Z score formula:z = (x - μ) / σLet us find Z score for x1 = 50.z1 = (x1 - μ) / σ = (50 - 42) / 14 = 8 / 14 = 0.57. The probability that x lies between 50 and 40 is 0.2734.

To find the probability of x between 50 and 40, we first need to find the Z scores for these two values. Using the Z score formula, we get z1 = 0.57 and z2 = -0.14. Next, we use the standard normal table to find the area between these two z scores. This gives us the probability that x lies between 50 and 40. Finally, we round the answer to four decimal places, which gives us 0.2734.

To find the probability of x between 50 and 40, we first need to find the Z scores for these two values. Using the Z score formula, we get z1 = 0.57 and z2 = -0.14.z1 = (x1 - μ) / σz2 = (x2 - μ) / σz1 = (50 - 42) / 14 = 8 / 14 = 0.57z2 = (40 - 42) / 14 = -0.14Next, we use the standard normal table to find the area between these two z scores. This gives us the probability that x lies between 50 and 40. To find the area between z1 and z2, we use the following formula:Area between z1 and z2 = P(z2 < Z < z1)P(z2 < Z < z1) = P(Z < z1) - P(Z < z2)P(z2 < Z < z1) = Φ(z1) - Φ(z2)Here, Φ(z) represents the area under the standard normal curve to the left of z. We can find the values of Φ(z) using the standard normal table. Substituting the values of z1 and z2, we get:P(50 ≤ x ≤ 40) = Φ(0.57) - Φ(-0.14)Now we can look at the standard normal table to find the values of Φ(0.57) and Φ(-0.14). We get:Φ(0.57) = 0.7186Φ(-0.14) = 0.4452Substituting in the values, we get:P(50 ≤ x ≤ 40) = Φ(0.57) - Φ(-0.14) = 0.7186 - 0.4452 = 0.2734Therefore, the probability that x lies between 50 and 40 is 0.2734. We can round this answer to four decimal places, which gives us the final answer.

To know more about Standard deviation visit :-

https://brainly.com/question/29115611

#SPJ11

Consider the sequence: an = ((3n+2)!) (3n-1)!) a. Find the first 6 terms of the sequence. b. Is the sequence bounded? c. Is the sequence increasing, decreasing, non-increasing, non-decreasing, or none of the above? d. According to the monotonic convergence theorem, does the series converge? e. If the sequence converges (by monotonic convergence or not), determine the value that the sequence converges to.

Answers

a. To find the first 6 terms of the sequence, we substitute the values of n from 1 to 6 into the given formula:

a1 = ((3(1)+2)!) / ((3(1)-1)!) = (5!) / (2!) = 120 / 2 = 60

a2 = ((3(2)+2)!) / ((3(2)-1)!) = (8!) / (5!) = (8 * 7 * 6 * 5!) / (2 * 1 * 5!) = 8 * 7 * 6 = 336

a3 = ((3(3)+2)!) / ((3(3)-1)!) = (11!) / (8!) = (11 * 10 * 9 * 8!) / (8!) = 11 * 10 * 9 = 990

a4 = ((3(4)+2)!) / ((3(4)-1)!) = (14!) / (11!) = (14 * 13 * 12 * 11!) / (11!) = 14 * 13 * 12 = 2184

a5 = ((3(5)+2)!) / ((3(5)-1)!) = (17!) / (14!) = (17 * 16 * 15 * 14!) / (14!) = 17 * 16 * 15 = 4080

a6 = ((3(6)+2)!) / ((3(6)-1)!) = (20!) / (17!) = (20 * 19 * 18 * 17!) / (17!) = 20 * 19 * 18 = 6840

The first 6 terms of the sequence are: 60, 336, 990, 2184, 4080, 6840.

b. To determine if the sequence is bounded, we need to examine if there exists a number M such that |an| ≤ M for all n. In this case, we can see that the terms of the sequence are factorial expressions, which grow very quickly as n increases. Therefore, the sequence is unbounded.

c. Since the sequence is unbounded, it does not exhibit a specific pattern of increase or decrease. Therefore, we cannot classify it as increasing, decreasing, non-increasing, or non-decreasing.

d. The sequence does not converge because it is unbounded.

e. As the sequence does not converge, there is no specific value that the sequence converges to.

To know more about Formula visit-

brainly.com/question/31062578

#SPJ11

$1,500 are deposited into an account with a 7% interest rate, compounded quarterly.

Find the accumulated amount after 5 years.

Hint: A=P(1+r/k)kt

Answers

Answer:

$2122.17

Step-by-step explanation:

Principal/Initial Value: P = $1500

Annual Interest Rate: r = 7% = 0.07

Compound Frequency: k = 4

Period of Time: t = 5

[tex]\displaystyle A=P\biggr(1+\frac{r}{k}\biggr)^{kt}\\\\A=1500\biggr(1+\frac{0.07}{4}\biggr)^{4(5)}\\\\A\approx\$2122.17[/tex]

Answer:

Step-by-step explanation:

the polynomial (x-2) is a factor of the polynomial 3x^2-8x 2

Answers

The polynomial (x-2) is not a factor of the polynomial [tex]3x^2[/tex] - 8x + 2. Therefore, the given statement is false.

To determine if the polynomial (x-2) is a factor of the polynomial 3x^2 - 8x + 2, we can check if substituting x = 2 into the polynomial yields a value of zero. If the result is zero, then (x-2) is a factor.

Substituting x = 2 into [tex]3x^2[/tex] - 8x + 2, we get:

3(2)^2 - 8(2) + 2 = 12 - 16 + 2 = -2

Since the result is not zero, we can conclude that (x-2) is not a factor of the polynomial [tex]3x^2[/tex] - 8x + 2.

In general, for a polynomial (x-a) to be a factor of a polynomial f(x), substituting x = a into f(x) should result in zero. If the result is not zero, then (x-a) is not a factor of f(x).

Learn more about factor here:

https://brainly.com/question/14549998

#SPJ11


What is the Sample Skewness for the following numbers:

mean of 94 , median of 88, and standard deviation of 66.29?

Answers

To calculate the sample skewness, we need the mean, median, and standard deviation of a set of numbers. In this case, the given numbers have a mean of 94, a median of 88, and a standard deviation of 66.29.

Sample skewness is a measure of the asymmetry of a distribution. It indicates whether the data is skewed to the left or right.

To calculate the sample skewness, we can use the formula:

Skewness = 3 * (Mean - Median) / Standard Deviation

Substituting the given values into the formula:

Skewness = 3 * (94 - 88) / 66.29

Skewness = 0.0905

The sample skewness for the given numbers is 0.0905. Since the skewness is positive, it indicates that the distribution is slightly skewed to the right. This means that the tail of the distribution is longer on the right side, and there may be some outliers or extreme values pulling the distribution towards the right.

Learn more about median here:

https://brainly.com/question/11237736

#SPJ11








2. Form the differential equation by = 19eax eliminating constant a from y² show all steps.

Answers

To form the differential equation by = 19eax, we need to differentiate both sides of the equation with respect to x.

Differentiating both sides of the equation by = 19eax with respect to x using the chain rule, we have:

d(by)/dx = d(19eax)/dx

On the left side, we differentiate y with respect to x, which gives us dy/dx.

On the right side, we differentiate 19eax with respect to x. The derivative of 19eax with respect to x can be found using the constant multiple rule and the chain rule. The derivative of eax with respect to x is aeax, and multiplying it by the constant 19 gives us 19aeax.

Therefore, the differential equation is:

dy/dx = 19aeax

Now, to eliminate the constant a from the equation, we can use the given expression y². We substitute y² for (by)² in the differential equation:

(dy/dx)² = (19aeax)²

Simplifying further, we have:

(dy/dx)² = 361a²eax²

Now we have the differential equation in terms of y and x:

(dy/dx)² = 361a²eax²

It's important to note that this differential equation is specific to the given equation by = 19eax. If the expression or initial conditions change, the differential equation will be different.

To know more about expression visit-

brainly.com/question/32609097

#SPJ11

We want to test a claim about two population standard deviations or variances. We want to use the methods of this chapter. What conditions must be satisfied?

Answers

When testing a claim about two population standard deviations or variances, several conditions must be satisfied. These conditions include independence, normality, and homogeneity of variances.

Independence: The samples from each population must be independent of each other. This means that the observations within one sample should not influence the observations in the other sample. Independence can be ensured through random sampling or experimental design.

Normality: The populations from which the samples are drawn should be approximately normally distributed. This condition is important because the sampling distribution of the sample variances or standard deviations follows a chi-square distribution, which is based on the assumption of normality.

Homogeneity of Variances: The variances of the two populations should be equal (homogeneity of variances). This condition is necessary when conducting hypothesis tests or constructing confidence intervals for the difference between two population variances or standard deviations. One common test to assess homogeneity of variances is the F-test.

Learn more about standard deviations here:

https://brainly.com/question/13498201

#SPJ11

Let V be a finite dimensional complex inner product space with a basis B = {₁,..., Un}. Define a n × n matrix A whose i, j entry is given by (v₁, vj). Prove that (a) (5 points) Define the notion of a Hermitian matrix. (b) (3 points) Show that A is Hermitian (c) (5 points) We define (, ) on Cn via (x, y) = x¹ Ay. Show that A ([v]B, [W]B) A = (v, w) for all v, w € V (d) (7 points) Show that (, ) is an inner product on C". A (e) (2 points) Show that if B is an orthonormal basis, then the matrix A defined previously is the identity matrix.

Answers

According to the question Let V be a finite dimensional complex inner product space with a basis are as follows :

(a) A Hermitian matrix is a square matrix A whose complex conjugate transpose is equal to itself, i.e., A* = A, where A* denotes the conjugate transpose of A.

(b) To show that A is Hermitian, we need to show that A* = A. Let's calculate the conjugate transpose of A:

A* = [ (v₁, v₁) (v₁, v₂) ... (v₁, vn) ]

[ (v₂, v₁) (v₂, v₂) ... (v₂, vn) ]

[ ... ... ... ]

[ (vn, v₁) (vn, v₂) ... (vn, vn) ]

Now let's compare A* with A. We can see that the (i, j) entry of A* is the complex conjugate of the (j, i) entry of A. Since the inner product is conjugate linear in its first argument, we have (vᵢ, vⱼ) = (vⱼ, vᵢ)* for all i, j. Therefore, A* = A, and we conclude that A is Hermitian.

(c) We have defined the inner product (x, y) as (x, y) = xAy, where x and y are column vectors. Now let's express the vectors x and y in terms of the given bases:

x = [x₁, x₂, ..., xn] = [v]B

y = [y₁, y₂, ..., yn] = [w]B

Using the definition of matrix multiplication, we have:

A[x]B = A[v]B = [ (v, v₁), (v, v₂), ..., (v, vn) ]

= [x₁, x₂, ..., xn] = x

Similarly, A[y]B = y.

Now let's calculate the expression A[x]B * A[y]B:

A[x]B * A[y]B = [ (v, v₁), (v, v₂), ..., (v, vn) ] * [ (w, v₁), (w, v₂), ..., (w, vn) ]

= [ (v, v₁)(w, v₁) + (v, v₂)(w, v₂) + ... + (v, vn)*(w, vn) ]

= (v, w)

Therefore, A([v]B, [w]B)A = (v, w) for all v, w ∈ V.

(d) To show that (, ) is an inner product on Cn, we need to verify the properties of an inner product:

Conjugate Symmetry: (x, y) = (y, x)*

This property holds because A* = A, and taking the complex conjugate of a complex number twice gives back the original number.

Linearity in the First Argument: (ax + by, z) = a(x, z) + b(y, z) for all a, b ∈ C and x, y, z ∈ Cn

This property holds because matrix multiplication distributes over addition and scalar multiplication.

Positive Definiteness: (x, x) > 0 for all x ≠ 0

Since A is Hermitian, all diagonal entries (vᵢ, vᵢ) are real and non-negative. Therefore, the inner product is positive definite.

(e) If B is an orthonormal basis, then the inner product (vᵢ, vⱼ) is 1 if i = j, and 0 otherwise. This implies that the matrix A will have ones on the diagonal and zeros off the diagonal. In other words, A is the identity matrix.

To know more about orthonormal visit-

brainly.com/question/32304405

#SPJ11

A ladder is leaning against the side of a building. The ladder is 10 meters long and the angle between the ladder and the building is 18°. How far up the building does the ladder reach (to the nearest hundredth)?

Answers

The distance the ladder reached to the building is 32.36 metres.

How to find the side of a right triangle?

A ladder is leaning against the side of a building. The ladder is 10 meters long and the angle between the ladder and the building is 18°.

Therefore, the distance of the ladder to the building can be calculated using trigonometric ratios as follows:

Therefore,

sin 18 = opposite / hypotenuse

sin 18 = 10 / x

cross multiply

x = 10 / sin 18

x = 10 / 0.30901699437

x = 32.3624595469

Therefore,

distance of the ladder on the building = 32.36 metres

learn more on right triangle here: https://brainly.com/question/20964813

#SPJ1

4.1.5 the number of terms 2. Mokoena is p years old. His brother is twice his age. 2.1 How old is his brother? 2.2 How old will Mokoena be in 10 years? 2.3 How old was his brother 3 years ago? 2.4 What will their combined age be in q years time.​

Answers

Answer:

To answer the questions regarding Mokoena's and his brother's ages, we'll use the given information:

Mokoena is p years old.

His brother is twice his age.

2.1 How old is his brother?

Since his brother is twice Mokoena's age, his brother's age would be 2p.

2.2 How old will Mokoena be in 10 years?

To find Mokoena's age in 10 years, we add 10 to his current age: p + 10.

2.3 How old was his brother 3 years ago?

To find his brother's age 3 years ago, we subtract 3 from his brother's current age: 2p - 3.

2.4 What will their combined age be in q years' time?

To find their combined age in q years' time, we add q to the sum of their current ages: p + 2p + q = 3p + q.

Therefore, the answers are:

2.1 His brother's age is 2p.

2.2 Mokoena will be p + 10 years old in 10 years.

2.3 His brother was 2p - 3 years old 3 years ago.

2.4 Their combined age in q years' time will be 3p + q.

Step-by-step explanation:

ASM is one of United States' tallest skyscrapers and is one of the most exclusive properties in Connecticut. Piper, who just got her freedom from Litchfield correctional area, wants to stay at the topmost floor unit. She hears about two unoccupied units in a building with 7 floors and eight units per floor. What is the probability that there is a unoccupied unit on the topmost floor? (correct to 4 significant figures)

Answers

The required probability, corrected to 4 significant figures = 0.1250 ≈ 0.0298 (correct to 4 significant figures). Hence, the solution is 0.0298.

The probability that there is an unoccupied unit on the topmost floor is 0.0298 (correct to 4 significant figures).

Given, Number of floors = 7

Number of units per floor = 8

Total number of units

= 7 × 8

= 56

The probability of getting an unoccupied unit on the topmost floor = P(E)

Let's calculate the probability of getting an unoccupied unit on any floor using the complement of the probability of getting an occupied unit.

P(getting an unoccupied unit) = 1 - P(getting an occupied unit)

Probability of getting an occupied unit on any floor = 56/56

Probability of getting an unoccupied unit on any floor

= 1 - 56/56

= 0

Therefore, the probability of getting an unoccupied unit on the topmost floor, P(E) = Probability of getting an unoccupied unit on any floor on the topmost floor

P(E) = (1/8) × (1 - 0)

= 1/8

= 0.125

∴ The probability that there is an unoccupied unit on the topmost floor is 0.125.

Therefore, the required probability, corrected to 4 significant figures = 0.1250 ≈ 0.0298 (correct to 4 significant figures).

Hence, the solution is 0.0298.

Know more about probability here:

https://brainly.com/question/25839839

#SPJ11

the school store sells pencils for $0.30 each, hats for $14.50 each, and binders for $3.20 each. elena wants to buy 3 pencils, a hat, and 2 binders. how much will her total cost be?

Answers

To calculate Elena's total cost, we need to multiply the quantity of each item she wants to buy by its respective price and then sum up the individual costs which will come out to be $21.80.

Elena wants to buy 3 pencils, which cost $0.30 each, so the total cost of the pencils will be 3 * $0.30 = $0.90. She also wants to buy a hat, which costs $14.50.

Additionally, Elena wants to buy 2 binders, which cost $3.20 each, so the total cost of the binders will be 2 * $3.20 = $6.40. To find the total cost, we add up the costs of each item: $0.90 + $14.50 + $6.40 = $21.80.

Learn more about total cost here: brainly.com/question/30355738
#SPJ11

Dwight is packing for a vacation. He is deciding which books to bring. He wants to bring one book from each genre. Given he has 2 science fiction, 2 business ethics, and 7 beet farming books, how many unique combinations of books could be bring with him?

Answers

Dwight can bring a total of 28 unique combinations of books with him.

To determine the number of unique combinations, we need to calculate the product of the number of choices for each genre.

In this case, Dwight has 2 choices for science fiction, 2 choices for business ethics, and 7 choices for beet farming books.

To find the total number of unique combinations, we multiply the number of choices for each genre together: 2 × 2 × 7 = 28.

Here's a breakdown of how we arrive at this result.

For each science fiction book, Dwight can choose 1 out of 2 options.

Similarly, for each business ethics book, he can choose 1 out of 2 options.

Finally, for each beet farming book, he can choose 1 out of 7 options.

By multiplying these choices together, we account for all possible combinations of books that Dwight can bring.

Therefore, Dwight has 28 unique combinations of books he can bring on his vacation, ensuring he has one book from each genre.

Learn more about combinations here:

https://brainly.com/question/29595163

#SPJ11

The probability density function f of a continuous random variable X is given by

f(x)= {cx+3, −3≤x≤−2,
3−cx, 2≤x≤3
0, otherwise

(a) Compute c.
(b) Determine the cumulative distribution function of X.
(c) Compute P(−1

Answers

The cumulative distribution function (CDF) of X is given by F(x) = {0, , (c) is the main answer.

We are required to find P(−1 ≤ X ≤ 1)First, we need to find the CDF of X, that is F(x).

for x ≤ −3, 1/18 (c(x+3)^2 + 27), for −3 < x ≤ −2, 1/18 (c(x+3)^2 + 27) + 1/18 (9 − c(x+2)^2),

for −2 < x ≤ 2, 1/18 (c(x+3)^2 + 27) + 1/18 (9 − c(x+2)^2) + 1/18 (9 − c(3−x)^2), for 2 < x ≤ 3, 1, for x > 3.

Therefore, (c) is the main answer.

Learn more about function click here:

https://brainly.com/question/11624077

#SPJ11








The number of people in a community who became infected during an epidemic t weeks after its outbreak is given by the function f(t)=- 25,000 1+ ac -kt- where 25,000 people of the community are suscept

Answers

The given function is f(t) = -25,000(1 + ac - kt), where t represents the number of weeks after the outbreak of an epidemic and f(t) represents the number of people in a community who became infected during that time.

The function takes into account the initial population of 25,000 people, the susceptibility coefficient a, the contact coefficient c, and the recovery coefficient k.

In the function, the term (1 + ac - kt) represents the probability of an individual becoming infected at a specific time t. The coefficient a represents the proportion of susceptible individuals in the community, while c represents the rate of contact between susceptible and infected individuals. The coefficient k represents the recovery rate or the rate at which infected individuals stop being contagious.

By evaluating the function f(t) at a specific value of t, we can determine the number of people who became infected during the epidemic t weeks after its outbreak. The function accounts for the initial population, the susceptibility of individuals, the rate of contact, and the recovery rate to calculate the number of infections.

Learn more about functions here: brainly.com/question/31062578
#SPJ11

A sample of size 60 from a population having standard deviation o = 35 produced a mean of 245.00. The 95% confidence interval for the population mean (rounded to two decimal places) is:

Answers

The confidence interval for the population mean is approximately (236.17, 253.83) when rounded to two decimal places.

How to find the confidence interval?

Here we want the 95% confidence interval for the population mean, we need to use the formula for a confidence interval:

CI = x ± Z * (σ / sqrt(n))

Where the variables are:

CI is the confidence intervalx is the sample meanZ is the Z-score corresponding to the desired confidence level (95% confidence corresponds to a Z-score of 1.96)σ is the standard deviation of the populationn is the sample size

Given:

Sample mean (x) = 245.00

Standard deviation (σ) = 35

Sample size (n) = 60

Desired confidence level = 95%

Now, let's calculate the confidence interval:

CI = 245.00 ± 1.96 * (35 / √60)

CI = 245.00 ± 1.96 * (35 / 7.746)

CI = 245.00 ± 1.96 * 4.51

CI = 245.00 ± 8.83

Learn more about confidence intervals at:

https://brainly.com/question/15712887

#SPJ4

8. [5pts.] Find a solution for sec(30-15°) = csc(+25°)

Answers

The solution is sin(25) ≈ 0.4226 and cos(15) ≈ 0.9659.

Given, sec(30 - 15°) = csc(+25°)We know that

sec(30 - 15°) = sec(15) and csc(+25°) = csc(25)

So, the equation becomes sec(15) = csc(25)

Now, we know that sec(x) = 1/cos(x) and csc(x) = 1/sin(x).So, sec(15) = 1/cos(15) and csc(25) = 1/sin(25)

Therefore, 1/cos(15) = 1/sin(25)sin(25) = cos(15)sin(25) ≈ 0.4226cos(15) ≈ 0.9659Hence, the solution is sin(25) ≈ 0.4226 and cos(15) ≈ 0.9659Answer:So, the solution is sin(25) ≈ 0.4226 and cos(15) ≈ 0.9659.

To know more about sin visit:-

https://brainly.com/question/19213118

#SPJ11

In problems 1-3 find all prime ideals and maximal ideals in the given ring.

Answers

   In problems 1-3, we need to find all prime ideals and maximal ideals in the given rings. The answer will be divided into two paragraphs, with the first paragraph summarizing the answer and the second paragraph providing an explanation.

To find the prime ideals in a given ring, we need to look for proper ideals that satisfy the prime property. An ideal I in a ring R is prime if for any elements a and b in R, if their product ab belongs to I, then either a or b (or both) must belong to I. Prime ideals are important in ring theory as they exhibit similar properties to prime numbers in the context of integers.
Maximal ideals, on the other hand, are proper ideals that are not contained in any other proper ideals. In other words, an ideal M in a ring R is maximal if there are no proper ideals properly containing M. Maximal ideals are significant because they provide insights into the structure and properties of the ring.
To determine all prime ideals and maximal ideals in a given ring, we need to carefully analyze the properties of the ring, including its elements, operations, and any special properties or constraints imposed on the ring. By examining the ring'sstructure and applying the definitions of prime and maximal ideals, we can identify and classify the prime and maximal ideals present in the ring.

learn more about prime ideals here

https://brainly.com/question/32544892



#SPJ11

Other Questions
please explain fully or will dislike.We want to predict y-salaries for people with the same job title based on x1=months at job and x2-gender (coded as males=0, females-1) using the model: y=x+Bx + x + 3x1x + E Identify th If a United States Savings bond can be purchased for $28.50 and has a maturity value of $100 at the end of 19 years, what is the annual rate of return on the bond? In the following model, derive the autonomous consumption, and the investment, export, and import multipliers. T = 0 Yd = Y - T C = 500 + 0.3Yd I = 600 G = 0 X = 100 M = 300 + 0.2Y 2. If the tax function changed to T + 0.25Y, and government spending was G + 25, calculate equilibrium income in question 1. How does this figure compare to your previous answer, when government spending and taxes are zero? Individual Assignment (30 Marks)You have been asked to present a technical paper on effectiveness of digital financial services as apposed to traditional financial services in Namibia.a. Discuss the types of digital finances in Namibia and how they have affected the economy.b. Describe the impact of new technologies on the financial services industry.c. Logically, relate your discussion to howi) prudential regulation & supervision,ii) monetary policies & regulation has impacted the digital financial services industry of Namibia Aiko buys a bouquet of 12 flowers from a floral shop.Some of the flowers in his bouquet are roses, which cost $2.60 each, and some are asters, which cost $1.20 each.Altogether, Aiko's bouquet costs exactly $20.Complete the table and write a system of equations to represent the number of each type of flower in Aiko's bouquet. Subject - Enterprise Architecture, Reference PatternMany major industries have formed professional bodies that have developed and support reference architectures a set of patterns at each layer of the enterprise that describe roles and activities performed by most industry participantsConsider the proposition: "Reference architectures remove creativity from the enterprise architecture in companies that leverage them." Pick one side or the other of this argument and post your thoughts A project manager can calculate the projected budget to completion by using the.A. Budget forecastB. Cost performance indexC. Schedule performance indexD. Risk management plan Using a case company of your choice and with relevantexamples, examine its financial management function and demonstratethe essence of sound financial management to supplychain/procurement managers Question 1Historically in the U.S., which of the following has always been true?A. Yield on a 2-year corporate bond > yield on a 2-year Treasury bond.B. Yield on a 3-year corporate bond > yield on a 2-year corporate bond.C. Yield on a 3-year Treasury bond > yield on a 2-year Treasury bondD. Yield on a 3-year corporate bond > yield on a 5-year Treasury bond.E. All of the above.F. Statements (a) and (c) 6: Which is not a characteristic of the sample standard deviation? A It is always the square root of the variance. It is not applicable when data are continuous. C. It is affected by outliers. D. It i Which of the following is an example of triangular arbitrage initiation? a. buying a currency at one bank's ask and selling at another bank's bid, which is higher than the former bank's ask. b. buying Singapore dollars from a bank (quoted at $.55) that has quoted the South African rand (SAR)/Singapore dollar (S$) exchange rate at SAR2.50 when the spot rate for the rand is $.20. c. buying Singapore dollars from a bank (quoted at $.55) that has quoted the South African rand/Singapore dollar exchange rate at SAR3.00 when the spot rate for the rand is $.20. d. converting funds to a foreign currency and investing the funds overseas.---Can you please show me step by step how the answer C produced an arbitrage, what was the calculation??--- (q5) Which of the following is the area of the surface obtained by rotating the curve , about the x-axis? 1. What role does the matching concept play in accounting forbad debts?2. how a reserve account can be usedto move earnings across different periods. Please describe the two principal ways that a bank can manage the drain in liquidity in the liability side of the balance sheet. Carefully explain the benefits and costs of the two approaches. Why do retirement income conversion options exist? A. to provide you with a regular source of retirement income B. to spread out the tax burden associated with making withdrawals from a taxable registered plan C. to convert your tax deferred savings to taxable income D. to prevent you from cashing in your retirement account Create a complete international marking plan for one of the following companies: a Japanese company seeking to export rice to other Asian countries Italian automobile company seeking to sell fuel-efficient cars in Africa Information technology software company in India seeking to expand to Australia Spanish winery seeking to enter markets in Central America4. Make a list of the emerging trends in international marketing that you identify. Explain how they have appeared, or become less relevant in the past decade. What new trends do you believe will affect international marketing? Explain your answer. Should the management of volunteers require the application ofmanagement principles different from those used in managing paidemployees? Whydid Chinese bought US dollars in the Asian financial crisis of1997, and what was the economic logic behind this rational? Find all solutions of the equation in the interval [0, 21). 2 cos 0+3 = 2 Write your answer in radians in terms of . If there is more than one solution, separate them with commas. 0 = 0 = 0,0.... B Need to express that equation in rectangular coordinates.Express the equation in rectangular coordinates. (Use the variables x Submit Answer X= 8 wwwwwww. 3.T 2 /2 2